Solving Linear Transformations in R2 and R3

  • Thread starter Thread starter dlevanchuk
  • Start date Start date
Click For Summary
The discussion focuses on finding the results of a linear transformation T: R2 -> R3 defined by T(e1) = u1 and T(e2) = u2, where u1 = [1; 0; -1] and u2 = [2; 1; 0]. The user successfully computes T([1; 1]) and T([2; -1]) by constructing a transformation matrix B from u1 and u2, leading to the result T([1; 1]) = [3; 1; -1]. They express confusion about the underlying logic of their approach, questioning why their method worked. The explanation provided clarifies that the transformation matrix's columns correspond to the images of the basis vectors under T, reinforcing the principles of linear transformations and matrix multiplication. Understanding this relationship is crucial for grasping how linear transformations operate in different vector spaces.
dlevanchuk
Messages
29
Reaction score
0
What have I done?

Homework Statement


(linear transformation)
Let T: R2 -> R3 be a linear transformation such that T(e1) = u1 and T(e2) = u2, where u1 = [1; 0; -1] and u2 = [2; 1; 0]. Find each of the following:
T([1; 1]) and T([2; -1])

Homework Equations


The Attempt at a Solution


Here is the thing. I've been sitting with this problem for a good hour, and cannot figure out. I just started playing around with numbers, toss em around a little bit, and in the end I've got the correct answer... But can somebody explain me, WHY is this the right way of doing it?? lol

First, I looked at u1 and u2 as a matrix A = [1 2; 0 1; -1 0]. Then i did a familiar A*e1 = b1 and A*e2 = b2, where I got b1= [1; 0; -1] and b2 = [2; 1; 0].. (thinking out loud, every matrix multiplication is a transformation, so it must be relating somehow..)

I let b1 and b2 be matrix (i call it transformation matrix) B = [1 2; 0 1; -1 0]

then I decided to multiply this transformation matrix B by a vector that needs to be transformed from R2 to R3 [1; 1] and got
[1 2; 0 1; -1 0] * [1; 1] = [3; 1; -1]

By why in the name of Albert did it work?? lol did I have a correct logic through out the problem??
 
Last edited:
Physics news on Phys.org


Yes. If T is a linear transformation on vector space V, and \{e_1, e_2, ..., e_n\} is a basis for V, then the matrix representation of T, relative to this basis (and, if the range space is different, a specific basis for the range) just has T(e_1), T(e_2), etc. as columns.

To see why that is true, remember that we would always have e_1= 1e_1+ 0e_2+ ...+ 0e_n so in matrix multiplication, it would be represented by the column matrix \begin{bmatrix}1 \\ 0 \\ 0 \\ ... \\ 0\end{bmatrix}. Multiplying any matrix by that just gives the first column of the matrix. Similarly, e_2 would be represented by \begin{bmatrix}0 \\ 1\\ 0 \\...\\0\end{bmatrix}.
 
Question: A clock's minute hand has length 4 and its hour hand has length 3. What is the distance between the tips at the moment when it is increasing most rapidly?(Putnam Exam Question) Answer: Making assumption that both the hands moves at constant angular velocities, the answer is ## \sqrt{7} .## But don't you think this assumption is somewhat doubtful and wrong?

Similar threads

  • · Replies 4 ·
Replies
4
Views
3K
Replies
12
Views
2K
  • · Replies 3 ·
Replies
3
Views
2K
  • · Replies 7 ·
Replies
7
Views
2K
Replies
4
Views
2K
  • · Replies 2 ·
Replies
2
Views
2K
  • · Replies 10 ·
Replies
10
Views
3K
Replies
3
Views
2K
  • · Replies 8 ·
Replies
8
Views
3K
  • · Replies 1 ·
Replies
1
Views
2K